How Does Subset Proof in Abstract Algebra Work?

Click For Summary
The discussion centers on proving two statements regarding subsets in abstract algebra. The first statement shows that B is a subset of A union the intersection of B and the complement of A, which is established through case analysis on the elements of B. The second statement asserts that A equals B if there exists a subset X such that A union X equals B union X, with specific conditions on the complements of A and B. Participants express confusion about the properties of set differences and the manipulation of set identities. Clarifications are provided to enhance understanding of these concepts and the proof process.
ktheo
Messages
51
Reaction score
0

Homework Statement


Question 1. Let U be a universal set, A and B two subsets of U.
(1) Show that
B ⊆ A ∪ (B ∩ A^c).
(2) A = B if and only if there exists a subset X of U such that A ∪ X = B ∪ X and X\A^c = X\B^c.

The Attempt at a Solution



My attempt at a solution is as follows:

Part 1: Showing B ⊆ A ∪ (B ∩ A^c)

(A∪B)∩(A∪A^c)
(A∪B)∩(\bigcup)

Since A∪B are both in universe, it serves that B ⊆ A ∪ (B ∩ A^c).

Part 2
A = B if and only if there exists a subset X of U such that A ∪ X = B ∪ X and X\A^c = X\B^c.

So I claimed double inclusion proof here, letting X\inA

Case 1: X\inX

X\inA\cupX\RightarrowX\inB\cupX\RightarrowX\inB

Case 2: X\notinX\RightarrowX\inX\A^c\RightarrowX\inX\B^c\RightarrowX\inX or X\inB^c but X\notinX so X\inB

So I think the problem with this question is that I am not fully understanding the concept of the property known as the difference.
 
Physics news on Phys.org
ktheo said:
My attempt at a solution is as follows:

Part 1: Showing B ⊆ A ∪ (B ∩ A^c)

(A∪B)∩(A∪A^c)
(A∪B)∩(\bigcup)

Since A∪B are both in universe, it serves that B ⊆ A ∪ (B ∩ A^c).
I don't understand your proof. May I suggest something along these lines: if b \in B, and b \not\in A, then b \in B \cap A^c. Therefore...
So I think the problem with this question is that I am not fully understanding the concept of the property known as the difference.
You may find it helpful to use the following identity: A \setminus B = A \cap B^c for any sets A, B \subset U.
 
jbunniii said:
I don't understand your proof. May I suggest something along these lines: if b \in B, and b \not\in A, then b \in B \cap A^c. Therefore...
Therefore B\inB and B\inA? Is that what you're implying? I'm confused where B\inB comes into play. I'm not sure I know how I'm supposed to approach this then... I thought I was supposed to manipulate the side B into the right side using the property for AUA^c equal to the universe. Or am I supposed to do some sort of proof using inclusion... I kind of use A=B as the signal that I am supposed to do that. I'm sorry bear with me I am quite new to all this...
You may find it helpful to use the following identity: A \setminus B = A \cap B^c for any sets A, B \subset U.

When A and B in this case are already compliments, that makes no difference? I can still switch using that identity?
 
ktheo said:
When A and B in this case are already compliments, that makes no difference? I can still switch using that identity?
Yes, the identity is valid in all cases. If you have something like X \setminus A^c then that equals X \cap(A^c)^c = X \cap A.
 
Hi jbunni, could you check out my second attempt at question part 1:

So B⊆A∪(B∩A^c)

To show this, we can say that when x∈B, implies there exists an x∈A∪(B∩A^c)

We will let x∈B,

Case 1: x∈ A

x∈ A--->x∈ A or x∈(B∩A^c). So clearly, x∈A.

Case 2: x∉A

x∈A or x∈(B∩A^c). We have declared x∉A, so x∈B and x∈A^c. Now we note that X∈A^c is = to X∉A.

Thus proving that B⊆A∪(B∩A^c)
 
ktheo said:
Hi jbunni, could you check out my second attempt at question part 1:

So B⊆A∪(B∩A^c)

To show this, we can say that when x∈B, implies there exists an x∈A∪(B∩A^c)

We will let x∈B,

Case 1: x∈ A

x∈ A--->x∈ A or x∈(B∩A^c). So clearly, x∈A.

Case 2: x∉A

x∈A or x∈(B∩A^c). We have declared x∉A, so x∈B and x∈A^c. Now we note that X∈A^c is = to X∉A.

Thus proving that B⊆A∪(B∩A^c)

I think you have the right idea, but I would word it somewhat differently. See if this sounds cleaner to you:

Let x \in B. We consider two cases:

Case 1: x \in A. In this case x \in A \cup (B \cap A^c) because A \subset A \cup (B \cap A^c).

Case 2: x \not\in A. Then x \in (B \cap A^c). Therefore x \in A \cup (B \cap A^c), because (B \cap A^c) \subset A \cup (B \cap A^c)

In both cases, we have established that x \in B implies x \in A \cup (B \cap A^c). This shows that B \subset A \cup (B \cap A^c).
 
Question: A clock's minute hand has length 4 and its hour hand has length 3. What is the distance between the tips at the moment when it is increasing most rapidly?(Putnam Exam Question) Answer: Making assumption that both the hands moves at constant angular velocities, the answer is ## \sqrt{7} .## But don't you think this assumption is somewhat doubtful and wrong?

Similar threads

  • · Replies 2 ·
Replies
2
Views
1K
Replies
1
Views
2K
Replies
4
Views
2K
  • · Replies 8 ·
Replies
8
Views
2K
  • · Replies 6 ·
Replies
6
Views
1K
Replies
3
Views
2K
  • · Replies 23 ·
Replies
23
Views
888
  • · Replies 2 ·
Replies
2
Views
1K
  • · Replies 9 ·
Replies
9
Views
2K
  • · Replies 3 ·
Replies
3
Views
1K